To use the pool at City Gym, one must have a membership there. Sarah has a membership at City Gym. She must therefore...

shafieiava on March 21, 2020

B, C & D

Can someone explain why answer choices C and D are incorrect? Also, can someone break down/simplify the phrasing in the correct answer, answer choice B? In general what category or type of error in reasoning is in the stimulus? I thought it was a part-to-whole issue which as why I selected D. Thanks for your time.

Reply
Create a free account to read and take part in forum discussions.

Already have an account? log in

Skylar on March 21, 2020

@shafieiava, thanks for your question.

The passage is flawed because it uses the Necessary condition to conclude the Sufficient condition.

Let's start by breaking down the passage.
- We should notice the word "must," which is a Necessary indicator. This allows the first sentence of the passage to be diagrammed as: Pool -> Membership. The contrapositive of this is: not Membership -> not Pool.
- The next sentence tells us that Sarah has a membership. This gives us the Necessary condition in our original diagram. We know that there is nothing to conclude based on this alone.
- The last sentence mistakenly uses the existence of the N condition (Sarah's membership) to conclude the existence of the S condition (Sarah uses the pool). If this flaw is not clear to you, I would recommend reviewing the Sufficient & Necessary Conditions lesson.

Now let's look at the answer choices you mentioned:

(B) "treats a statement whose truth is required for the conclusion to be true as though it were a statement whose truth ensures that the conclusion is true."

By definition, a Necessary condition is a condition that is required for the conclusion to be true, while a Sufficient condition ensures that the conclusion is true. Therefore, this answer choice is essentially a fancy way of saying that the passage treats a Necessary condition as a Sufficient condition. This is the flaw we identified in the passage, so (B) is correct. The key here is not to get thrown off by the wording of the answer choice.

(C) "presumes that one or the other of two alternatives must be the case without establishing that no other alternative is possible."

This is incorrect because the passage does not list a limited set of alternatives. (C) could be the correct answer if the passage read: "Sarah is using the pool at City Gym, so she either broke in or is using a guest pass." This hypothetical passage does consider the alternative that Sarah herself has a membership. Moreover, it does not limit the alternatives by saying that there are no other possible explanations for Sarah's usage or by saying that breaking in or using a guest pass are the only ways to use the pool. This is different from our original passage, which indicates that the only way to use the pool is with a membership.

(D) "concludes that a person has a certain attribute because that person belongs to a group most of whose members have that attribute."

This refers to a type of whole-to-part flaw (AKA the fallacy of division), where we conclude that an individual has a certain characteristic just because the whole has that characteristic. This cannot be the correct flaw because the passage never discusses a group of people.

Does that make sense? Please let us know if you have any other questions!